LSAT and Law School Admissions Forum

Get expert LSAT preparation and law school admissions advice from PowerScore Test Preparation.

 Arindom
  • Posts: 76
  • Joined: Apr 11, 2016
|
#25176
Hi,

I had a hard time with this question. The Supervisor says that our next budget will probably be approved as the previous five proposals were all turned down. Then, why is ans choice C correct? The argument isn't saying that the next budget proposal will be turned down.

Thanks.

- Arindom
 David Boyle
PowerScore Staff
  • PowerScore Staff
  • Posts: 836
  • Joined: Jun 07, 2013
|
#25384
Arindom wrote:Hi,

I had a hard time with this question. The Supervisor says that our next budget will probably be approved as the previous five proposals were all turned down. Then, why is ans choice C correct? The argument isn't saying that the next budget proposal will be turned down.

Thanks.

- Arindom

Hello Arindom,

True, the argument isn't saying that. But answer C is really addressing the possibility that the next proposal will be turned down...or approved. If it's not turned down, it's approved, presumably, so the "likelihood" of being turned down is inverse to the likelihood of being approved. Even if the last five were turned down, that doesn't magically assure that the next one will be approved.

Hope this helps,
David
 akanshalsat
  • Posts: 104
  • Joined: Dec 20, 2017
|
#48635
Hello,

Shouldn't A be correct since the supervisor thinks that if half of all budget proposals are approved, and 5 are already not approved, then the next five will be approved, and he says that he feels the next one will be approved (which would be part of the next five)?

I feel that C is talking about it being turned down, but the supervisor isnt thinking the next budget will be turned down, he's thinking it'll be approved?
 Adam Tyson
PowerScore Staff
  • PowerScore Staff
  • Posts: 5153
  • Joined: Apr 14, 2011
|
#49070
"Affecting the likelihood" of something doesn't mean it will make it happen, akanshalsat. It means it will have an influence on whether or not it happens, and that influence could be positive or negative.

Our author doesn't have to assume that all of the next five proposals will be approved in order to claim that the next one probably will. All he's saying is something like "our luck has got to change soon". That's like flipping a coin five times and getting tails every time, and then claiming that it will probably (more likely than not) be heads next time because it has been just tails up to this point. But what are the actual odds of it coming up heads next time? 50/50, just like every time you flip the coin! Getting 5 tails in a row, or 500 in a row, does not do anything to change the odds of it coming up heads next time. That's the flaw here!

Another problem with answer A is that the author doesn't assume anything is guaranteed, just that the probability has been affected. He says the next budget proposal will probably be approved, not that it is certain to be.

May the odds be ever in your favor!
 Mariam
  • Posts: 21
  • Joined: Apr 04, 2020
|
#76081
Hello- I have a question regarding question 24 as featured in the "Flawed Assumption Questions" module in Lesson 5 on the online student center. I chose answer choice A because I thought that with a 50% chance, if the author is arguing that because the last 5 proposals were denied then the next one should be approved, the only way that this argument works is if the author assumes that there is a total of 10 proposals and with a 50% chance 5 must be approved and 5 denied. I see how answer choice C is correct, but someone please elaborate as to why A is incorrect. I just want to understand where I went wrong so I can hopefully avoid this in the future. Thank you so much in advance!!
 Rachael Wilkenfeld
PowerScore Staff
  • PowerScore Staff
  • Posts: 1358
  • Joined: Dec 15, 2011
|
#76107
Hi Mariam,

The reason answer choice (a) is incorrect here is because an assumption question asks what is REQUIRED to prove our conclusion. Our conclusion here is that the proposal will probably be approved. That's not a particularly strongly worded conclusion. Answer choice (a) is far too broad to be required. It not only states that the proposal will be approved, but that the four proposals after it will be approved as well. That's much more than we need to show that the proposal will probably be approved. We don't need to know anything about the four following proposals at all to reach our conclusion.

Remember with assumptions that you are looking for the answer that is absolutely required in order for the conclusion to be true. When you see extraneous information in an answer choice that does not impact the conclusion, it's likely incorrect for an assumption question. It's telling you more than you are required to have.

Hope that helps!
Rachael

Get the most out of your LSAT Prep Plus subscription.

Analyze and track your performance with our Testing and Analytics Package.